Finding the constrain of integralML-inequality for real integralsEvaluating the integral $int_0^infty fracsin x x ,mathrm dx = frac pi 2$?Proving the inequality of an integral applied to a sumFind values so that integral is a bounded operatorProblem in series expansionSolving the improper integral $1/(x^a+y^b)$Convergence of the integral: $I_alpha =int _0^infty left(frace^-alpha xsqrtxright):dx$definition of Fourier transform questionsHow to calculate $iint_D (x^2 + y^2) , dxdy$ with $D = (fracx2)^2 + (fracy3)^2 leq 1$?Integral transform with reciprocal complex exponential functions?permute double integral and limit

What happens if you do emergency landing on a US base in middle of the ocean?

Why does the Schrödinger equation work so well for the Hydrogen atom despite the relativistic boundary at the nucleus?

Why did a party with more votes get fewer seats in the 2019 European Parliament election in Denmark?

Are there cubesats in GEO?

What happens to foam insulation board after you pour concrete slab?

How could a government be implemented in a virtual reality?

Why did Hela need Heimdal's sword?

When writing an error prompt, should we end the sentence with a exclamation mark or a dot?

Reading two lines in piano

What's the correct term for a waitress in the Middle Ages?

Movie where a boy is transported into the future by an alien spaceship

How hard would it be to convert a glider into an powered electric aircraft?

You've spoiled/damaged the card

Sharing one invocation list between multiple events on the same object in C#

Java guess the number

How bad would a partial hash leak be, realistically?

How to supress loops in a digraph?

Importance sampling estimation of power function

Do manufacturers try make their components as close to ideal ones as possible?

X-shaped crossword

What is the purpose of building foundations?

Is the decompression of compressed and encrypted data without decryption also theoretically impossible?

How is it possible that Gollum speaks Westron?

What can plausibly explain many of my very long and low-tech bridges?



Finding the constrain of integral


ML-inequality for real integralsEvaluating the integral $int_0^infty fracsin x x ,mathrm dx = frac pi 2$?Proving the inequality of an integral applied to a sumFind values so that integral is a bounded operatorProblem in series expansionSolving the improper integral $1/(x^a+y^b)$Convergence of the integral: $I_alpha =int _0^infty left(frace^-alpha xsqrtxright):dx$definition of Fourier transform questionsHow to calculate $iint_D (x^2 + y^2) , dxdy$ with $D = (fracx2)^2 + (fracy3)^2 leq 1$?Integral transform with reciprocal complex exponential functions?permute double integral and limit













2












$begingroup$


$ f_n(x) = left(4x-x^2right)^n$



I'm trying to prove that $int _0^4:f_n(x),dx < 4^n+1$



When I'm doing a general integral to $n > 1$, I'm getting an expression that equals to zero when $X = 4$ or $X = 0$.



This is the expression:



$$fracleft(4x-x^2right)^n+1(4-2X)(n+1) $$



What am I missing? Because it doesn't not make sense that every integral with $n > 1$ is equal to zero










share|cite|improve this question











$endgroup$
















    2












    $begingroup$


    $ f_n(x) = left(4x-x^2right)^n$



    I'm trying to prove that $int _0^4:f_n(x),dx < 4^n+1$



    When I'm doing a general integral to $n > 1$, I'm getting an expression that equals to zero when $X = 4$ or $X = 0$.



    This is the expression:



    $$fracleft(4x-x^2right)^n+1(4-2X)(n+1) $$



    What am I missing? Because it doesn't not make sense that every integral with $n > 1$ is equal to zero










    share|cite|improve this question











    $endgroup$














      2












      2








      2


      1



      $begingroup$


      $ f_n(x) = left(4x-x^2right)^n$



      I'm trying to prove that $int _0^4:f_n(x),dx < 4^n+1$



      When I'm doing a general integral to $n > 1$, I'm getting an expression that equals to zero when $X = 4$ or $X = 0$.



      This is the expression:



      $$fracleft(4x-x^2right)^n+1(4-2X)(n+1) $$



      What am I missing? Because it doesn't not make sense that every integral with $n > 1$ is equal to zero










      share|cite|improve this question











      $endgroup$




      $ f_n(x) = left(4x-x^2right)^n$



      I'm trying to prove that $int _0^4:f_n(x),dx < 4^n+1$



      When I'm doing a general integral to $n > 1$, I'm getting an expression that equals to zero when $X = 4$ or $X = 0$.



      This is the expression:



      $$fracleft(4x-x^2right)^n+1(4-2X)(n+1) $$



      What am I missing? Because it doesn't not make sense that every integral with $n > 1$ is equal to zero







      analysis






      share|cite|improve this question















      share|cite|improve this question













      share|cite|improve this question




      share|cite|improve this question








      edited 8 hours ago









      Bernard

      127k743120




      127k743120










      asked 8 hours ago









      DanielDaniel

      111




      111




















          3 Answers
          3






          active

          oldest

          votes


















          4












          $begingroup$

          Hint:



          $4x-x^2$ is a quadratic polynomial , with a negative leading coefficient. Furthermore, it vanishes for $x=0$ and $x=4$. Can you find it (absolute) maximum?



          Once you've found this maximum, you can use the mean value inequality for integrals.






          share|cite|improve this answer









          $endgroup$












          • $begingroup$
            Of course, you can estimate this by the maximum! very nice approach. I like this argument. This is probably how the question was meant to be done ^^
            $endgroup$
            – Wesley Strik
            8 hours ago



















          1












          $begingroup$

          You cannot just blindly apply the reverse power rule. Remember the chain rule? You have a function within another function.



          Consider the case when n=2



          $ int_0^4 (4x + x^2)^2 dx = int_0^4 16x^2 + 8x^3 + x^4 dx$






          share|cite|improve this answer









          $endgroup$












          • $begingroup$
            Would it be better for him to use the binomial expansion, you think?
            $endgroup$
            – Wesley Strik
            8 hours ago






          • 1




            $begingroup$
            That would be an excellent approach. I was also thinking that you could attempt to find a pattern and then use induction to prove it.
            $endgroup$
            – agentnola
            8 hours ago










          • $begingroup$
            I started working out the binomial theorem approach, you can switch the order of integration and summation and then integrate the power of $x$, this should not be too hard, although it will get a bit messy: $$int _x=0^4:f_n(x),dx =int _x=0^4: (4x-x^2 )^n,dx =int _x=0^4: sum_k=0^n binomnk (4x)^k cdot (-x^2)^n-k,dx$$
            $endgroup$
            – Wesley Strik
            8 hours ago











          • $begingroup$
            Now reduce everything to some power of $x$, move the integral inside and then we hope to get something pretty?
            $endgroup$
            – Wesley Strik
            8 hours ago


















          0












          $begingroup$

          Below is a suggestion for a very tedious route, but I wanted to post it anyway. This is probably not how the question is meant to be done.



          Observe how by the binomial theorem we have that:
          $$I=int _x=0^4:f_n(x),dx =int _x=0^4: (4x-x^2 )^n,dx =int _x=0^4: sum_k=0^n binomnk (4x)^k cdot (-x^2)^n-k,dx$$
          We make this look a bit prettier:
          $$I=int _x=0^4: sum_k=0^n binomnk 4^k cdot (-1)^n-kx^2n-k,dx$$



          We may switch the order of integration and summation since this concerns polynomials (what I mean is that they are of finite order), we compute:
          $$I=sum_k=0^n binomnk frac12n-k+1 4^k cdot (-1)^n-k4^2n-k+1-0$$
          We collect some terms:
          $$I=sum_k=0^n binomnk frac12n-k+1 4^2n+1 cdot (-1)^n-k$$
          Using combinatorial formulae and some help from Wolfram Alpha, this can be exactly computed to be:
          $$ I =fracsqrt pi cdot n! cdot 2^2n+1(n+frac12)!=fracsqrt pi cdot n!(n+frac12)! cdot 4^n + frac12$$
          We then need to prove by induction that $fracsqrt pi cdot n!(n+frac12)! > sqrt4$ for positive $n$.



          Yes. This is a lot of hard work. You could also just take the approach that was posted by Bernard and observe that $4x-x^2$ has a global maximum on the interval $[0,4]$ (the begin and endpoint are zeros of this polynomial), this maximum by symmetry occurs at $x=2$. The function has maximum value $8-4=4$. Observe then by the estimation lemma that $M =max_x in [0,4]((4x-x^2)^n)=4^n$, and the length of our path is $4$. We get:
          $$ int _x=0^4 f_n dx leq M L = 4^n cdot 4 = 4^n+1 $$



          Also see: ML-inequality for real integrals






          share|cite|improve this answer









          $endgroup$













            Your Answer








            StackExchange.ready(function()
            var channelOptions =
            tags: "".split(" "),
            id: "69"
            ;
            initTagRenderer("".split(" "), "".split(" "), channelOptions);

            StackExchange.using("externalEditor", function()
            // Have to fire editor after snippets, if snippets enabled
            if (StackExchange.settings.snippets.snippetsEnabled)
            StackExchange.using("snippets", function()
            createEditor();
            );

            else
            createEditor();

            );

            function createEditor()
            StackExchange.prepareEditor(
            heartbeatType: 'answer',
            autoActivateHeartbeat: false,
            convertImagesToLinks: true,
            noModals: true,
            showLowRepImageUploadWarning: true,
            reputationToPostImages: 10,
            bindNavPrevention: true,
            postfix: "",
            imageUploader:
            brandingHtml: "Powered by u003ca class="icon-imgur-white" href="https://imgur.com/"u003eu003c/au003e",
            contentPolicyHtml: "User contributions licensed under u003ca href="https://creativecommons.org/licenses/by-sa/3.0/"u003ecc by-sa 3.0 with attribution requiredu003c/au003e u003ca href="https://stackoverflow.com/legal/content-policy"u003e(content policy)u003c/au003e",
            allowUrls: true
            ,
            noCode: true, onDemand: true,
            discardSelector: ".discard-answer"
            ,immediatelyShowMarkdownHelp:true
            );



            );













            draft saved

            draft discarded


















            StackExchange.ready(
            function ()
            StackExchange.openid.initPostLogin('.new-post-login', 'https%3a%2f%2fmath.stackexchange.com%2fquestions%2f3247910%2ffinding-the-constrain-of-integral%23new-answer', 'question_page');

            );

            Post as a guest















            Required, but never shown

























            3 Answers
            3






            active

            oldest

            votes








            3 Answers
            3






            active

            oldest

            votes









            active

            oldest

            votes






            active

            oldest

            votes









            4












            $begingroup$

            Hint:



            $4x-x^2$ is a quadratic polynomial , with a negative leading coefficient. Furthermore, it vanishes for $x=0$ and $x=4$. Can you find it (absolute) maximum?



            Once you've found this maximum, you can use the mean value inequality for integrals.






            share|cite|improve this answer









            $endgroup$












            • $begingroup$
              Of course, you can estimate this by the maximum! very nice approach. I like this argument. This is probably how the question was meant to be done ^^
              $endgroup$
              – Wesley Strik
              8 hours ago
















            4












            $begingroup$

            Hint:



            $4x-x^2$ is a quadratic polynomial , with a negative leading coefficient. Furthermore, it vanishes for $x=0$ and $x=4$. Can you find it (absolute) maximum?



            Once you've found this maximum, you can use the mean value inequality for integrals.






            share|cite|improve this answer









            $endgroup$












            • $begingroup$
              Of course, you can estimate this by the maximum! very nice approach. I like this argument. This is probably how the question was meant to be done ^^
              $endgroup$
              – Wesley Strik
              8 hours ago














            4












            4








            4





            $begingroup$

            Hint:



            $4x-x^2$ is a quadratic polynomial , with a negative leading coefficient. Furthermore, it vanishes for $x=0$ and $x=4$. Can you find it (absolute) maximum?



            Once you've found this maximum, you can use the mean value inequality for integrals.






            share|cite|improve this answer









            $endgroup$



            Hint:



            $4x-x^2$ is a quadratic polynomial , with a negative leading coefficient. Furthermore, it vanishes for $x=0$ and $x=4$. Can you find it (absolute) maximum?



            Once you've found this maximum, you can use the mean value inequality for integrals.







            share|cite|improve this answer












            share|cite|improve this answer



            share|cite|improve this answer










            answered 8 hours ago









            BernardBernard

            127k743120




            127k743120











            • $begingroup$
              Of course, you can estimate this by the maximum! very nice approach. I like this argument. This is probably how the question was meant to be done ^^
              $endgroup$
              – Wesley Strik
              8 hours ago

















            • $begingroup$
              Of course, you can estimate this by the maximum! very nice approach. I like this argument. This is probably how the question was meant to be done ^^
              $endgroup$
              – Wesley Strik
              8 hours ago
















            $begingroup$
            Of course, you can estimate this by the maximum! very nice approach. I like this argument. This is probably how the question was meant to be done ^^
            $endgroup$
            – Wesley Strik
            8 hours ago





            $begingroup$
            Of course, you can estimate this by the maximum! very nice approach. I like this argument. This is probably how the question was meant to be done ^^
            $endgroup$
            – Wesley Strik
            8 hours ago












            1












            $begingroup$

            You cannot just blindly apply the reverse power rule. Remember the chain rule? You have a function within another function.



            Consider the case when n=2



            $ int_0^4 (4x + x^2)^2 dx = int_0^4 16x^2 + 8x^3 + x^4 dx$






            share|cite|improve this answer









            $endgroup$












            • $begingroup$
              Would it be better for him to use the binomial expansion, you think?
              $endgroup$
              – Wesley Strik
              8 hours ago






            • 1




              $begingroup$
              That would be an excellent approach. I was also thinking that you could attempt to find a pattern and then use induction to prove it.
              $endgroup$
              – agentnola
              8 hours ago










            • $begingroup$
              I started working out the binomial theorem approach, you can switch the order of integration and summation and then integrate the power of $x$, this should not be too hard, although it will get a bit messy: $$int _x=0^4:f_n(x),dx =int _x=0^4: (4x-x^2 )^n,dx =int _x=0^4: sum_k=0^n binomnk (4x)^k cdot (-x^2)^n-k,dx$$
              $endgroup$
              – Wesley Strik
              8 hours ago











            • $begingroup$
              Now reduce everything to some power of $x$, move the integral inside and then we hope to get something pretty?
              $endgroup$
              – Wesley Strik
              8 hours ago















            1












            $begingroup$

            You cannot just blindly apply the reverse power rule. Remember the chain rule? You have a function within another function.



            Consider the case when n=2



            $ int_0^4 (4x + x^2)^2 dx = int_0^4 16x^2 + 8x^3 + x^4 dx$






            share|cite|improve this answer









            $endgroup$












            • $begingroup$
              Would it be better for him to use the binomial expansion, you think?
              $endgroup$
              – Wesley Strik
              8 hours ago






            • 1




              $begingroup$
              That would be an excellent approach. I was also thinking that you could attempt to find a pattern and then use induction to prove it.
              $endgroup$
              – agentnola
              8 hours ago










            • $begingroup$
              I started working out the binomial theorem approach, you can switch the order of integration and summation and then integrate the power of $x$, this should not be too hard, although it will get a bit messy: $$int _x=0^4:f_n(x),dx =int _x=0^4: (4x-x^2 )^n,dx =int _x=0^4: sum_k=0^n binomnk (4x)^k cdot (-x^2)^n-k,dx$$
              $endgroup$
              – Wesley Strik
              8 hours ago











            • $begingroup$
              Now reduce everything to some power of $x$, move the integral inside and then we hope to get something pretty?
              $endgroup$
              – Wesley Strik
              8 hours ago













            1












            1








            1





            $begingroup$

            You cannot just blindly apply the reverse power rule. Remember the chain rule? You have a function within another function.



            Consider the case when n=2



            $ int_0^4 (4x + x^2)^2 dx = int_0^4 16x^2 + 8x^3 + x^4 dx$






            share|cite|improve this answer









            $endgroup$



            You cannot just blindly apply the reverse power rule. Remember the chain rule? You have a function within another function.



            Consider the case when n=2



            $ int_0^4 (4x + x^2)^2 dx = int_0^4 16x^2 + 8x^3 + x^4 dx$







            share|cite|improve this answer












            share|cite|improve this answer



            share|cite|improve this answer










            answered 8 hours ago









            agentnolaagentnola

            389




            389











            • $begingroup$
              Would it be better for him to use the binomial expansion, you think?
              $endgroup$
              – Wesley Strik
              8 hours ago






            • 1




              $begingroup$
              That would be an excellent approach. I was also thinking that you could attempt to find a pattern and then use induction to prove it.
              $endgroup$
              – agentnola
              8 hours ago










            • $begingroup$
              I started working out the binomial theorem approach, you can switch the order of integration and summation and then integrate the power of $x$, this should not be too hard, although it will get a bit messy: $$int _x=0^4:f_n(x),dx =int _x=0^4: (4x-x^2 )^n,dx =int _x=0^4: sum_k=0^n binomnk (4x)^k cdot (-x^2)^n-k,dx$$
              $endgroup$
              – Wesley Strik
              8 hours ago











            • $begingroup$
              Now reduce everything to some power of $x$, move the integral inside and then we hope to get something pretty?
              $endgroup$
              – Wesley Strik
              8 hours ago
















            • $begingroup$
              Would it be better for him to use the binomial expansion, you think?
              $endgroup$
              – Wesley Strik
              8 hours ago






            • 1




              $begingroup$
              That would be an excellent approach. I was also thinking that you could attempt to find a pattern and then use induction to prove it.
              $endgroup$
              – agentnola
              8 hours ago










            • $begingroup$
              I started working out the binomial theorem approach, you can switch the order of integration and summation and then integrate the power of $x$, this should not be too hard, although it will get a bit messy: $$int _x=0^4:f_n(x),dx =int _x=0^4: (4x-x^2 )^n,dx =int _x=0^4: sum_k=0^n binomnk (4x)^k cdot (-x^2)^n-k,dx$$
              $endgroup$
              – Wesley Strik
              8 hours ago











            • $begingroup$
              Now reduce everything to some power of $x$, move the integral inside and then we hope to get something pretty?
              $endgroup$
              – Wesley Strik
              8 hours ago















            $begingroup$
            Would it be better for him to use the binomial expansion, you think?
            $endgroup$
            – Wesley Strik
            8 hours ago




            $begingroup$
            Would it be better for him to use the binomial expansion, you think?
            $endgroup$
            – Wesley Strik
            8 hours ago




            1




            1




            $begingroup$
            That would be an excellent approach. I was also thinking that you could attempt to find a pattern and then use induction to prove it.
            $endgroup$
            – agentnola
            8 hours ago




            $begingroup$
            That would be an excellent approach. I was also thinking that you could attempt to find a pattern and then use induction to prove it.
            $endgroup$
            – agentnola
            8 hours ago












            $begingroup$
            I started working out the binomial theorem approach, you can switch the order of integration and summation and then integrate the power of $x$, this should not be too hard, although it will get a bit messy: $$int _x=0^4:f_n(x),dx =int _x=0^4: (4x-x^2 )^n,dx =int _x=0^4: sum_k=0^n binomnk (4x)^k cdot (-x^2)^n-k,dx$$
            $endgroup$
            – Wesley Strik
            8 hours ago





            $begingroup$
            I started working out the binomial theorem approach, you can switch the order of integration and summation and then integrate the power of $x$, this should not be too hard, although it will get a bit messy: $$int _x=0^4:f_n(x),dx =int _x=0^4: (4x-x^2 )^n,dx =int _x=0^4: sum_k=0^n binomnk (4x)^k cdot (-x^2)^n-k,dx$$
            $endgroup$
            – Wesley Strik
            8 hours ago













            $begingroup$
            Now reduce everything to some power of $x$, move the integral inside and then we hope to get something pretty?
            $endgroup$
            – Wesley Strik
            8 hours ago




            $begingroup$
            Now reduce everything to some power of $x$, move the integral inside and then we hope to get something pretty?
            $endgroup$
            – Wesley Strik
            8 hours ago











            0












            $begingroup$

            Below is a suggestion for a very tedious route, but I wanted to post it anyway. This is probably not how the question is meant to be done.



            Observe how by the binomial theorem we have that:
            $$I=int _x=0^4:f_n(x),dx =int _x=0^4: (4x-x^2 )^n,dx =int _x=0^4: sum_k=0^n binomnk (4x)^k cdot (-x^2)^n-k,dx$$
            We make this look a bit prettier:
            $$I=int _x=0^4: sum_k=0^n binomnk 4^k cdot (-1)^n-kx^2n-k,dx$$



            We may switch the order of integration and summation since this concerns polynomials (what I mean is that they are of finite order), we compute:
            $$I=sum_k=0^n binomnk frac12n-k+1 4^k cdot (-1)^n-k4^2n-k+1-0$$
            We collect some terms:
            $$I=sum_k=0^n binomnk frac12n-k+1 4^2n+1 cdot (-1)^n-k$$
            Using combinatorial formulae and some help from Wolfram Alpha, this can be exactly computed to be:
            $$ I =fracsqrt pi cdot n! cdot 2^2n+1(n+frac12)!=fracsqrt pi cdot n!(n+frac12)! cdot 4^n + frac12$$
            We then need to prove by induction that $fracsqrt pi cdot n!(n+frac12)! > sqrt4$ for positive $n$.



            Yes. This is a lot of hard work. You could also just take the approach that was posted by Bernard and observe that $4x-x^2$ has a global maximum on the interval $[0,4]$ (the begin and endpoint are zeros of this polynomial), this maximum by symmetry occurs at $x=2$. The function has maximum value $8-4=4$. Observe then by the estimation lemma that $M =max_x in [0,4]((4x-x^2)^n)=4^n$, and the length of our path is $4$. We get:
            $$ int _x=0^4 f_n dx leq M L = 4^n cdot 4 = 4^n+1 $$



            Also see: ML-inequality for real integrals






            share|cite|improve this answer









            $endgroup$

















              0












              $begingroup$

              Below is a suggestion for a very tedious route, but I wanted to post it anyway. This is probably not how the question is meant to be done.



              Observe how by the binomial theorem we have that:
              $$I=int _x=0^4:f_n(x),dx =int _x=0^4: (4x-x^2 )^n,dx =int _x=0^4: sum_k=0^n binomnk (4x)^k cdot (-x^2)^n-k,dx$$
              We make this look a bit prettier:
              $$I=int _x=0^4: sum_k=0^n binomnk 4^k cdot (-1)^n-kx^2n-k,dx$$



              We may switch the order of integration and summation since this concerns polynomials (what I mean is that they are of finite order), we compute:
              $$I=sum_k=0^n binomnk frac12n-k+1 4^k cdot (-1)^n-k4^2n-k+1-0$$
              We collect some terms:
              $$I=sum_k=0^n binomnk frac12n-k+1 4^2n+1 cdot (-1)^n-k$$
              Using combinatorial formulae and some help from Wolfram Alpha, this can be exactly computed to be:
              $$ I =fracsqrt pi cdot n! cdot 2^2n+1(n+frac12)!=fracsqrt pi cdot n!(n+frac12)! cdot 4^n + frac12$$
              We then need to prove by induction that $fracsqrt pi cdot n!(n+frac12)! > sqrt4$ for positive $n$.



              Yes. This is a lot of hard work. You could also just take the approach that was posted by Bernard and observe that $4x-x^2$ has a global maximum on the interval $[0,4]$ (the begin and endpoint are zeros of this polynomial), this maximum by symmetry occurs at $x=2$. The function has maximum value $8-4=4$. Observe then by the estimation lemma that $M =max_x in [0,4]((4x-x^2)^n)=4^n$, and the length of our path is $4$. We get:
              $$ int _x=0^4 f_n dx leq M L = 4^n cdot 4 = 4^n+1 $$



              Also see: ML-inequality for real integrals






              share|cite|improve this answer









              $endgroup$















                0












                0








                0





                $begingroup$

                Below is a suggestion for a very tedious route, but I wanted to post it anyway. This is probably not how the question is meant to be done.



                Observe how by the binomial theorem we have that:
                $$I=int _x=0^4:f_n(x),dx =int _x=0^4: (4x-x^2 )^n,dx =int _x=0^4: sum_k=0^n binomnk (4x)^k cdot (-x^2)^n-k,dx$$
                We make this look a bit prettier:
                $$I=int _x=0^4: sum_k=0^n binomnk 4^k cdot (-1)^n-kx^2n-k,dx$$



                We may switch the order of integration and summation since this concerns polynomials (what I mean is that they are of finite order), we compute:
                $$I=sum_k=0^n binomnk frac12n-k+1 4^k cdot (-1)^n-k4^2n-k+1-0$$
                We collect some terms:
                $$I=sum_k=0^n binomnk frac12n-k+1 4^2n+1 cdot (-1)^n-k$$
                Using combinatorial formulae and some help from Wolfram Alpha, this can be exactly computed to be:
                $$ I =fracsqrt pi cdot n! cdot 2^2n+1(n+frac12)!=fracsqrt pi cdot n!(n+frac12)! cdot 4^n + frac12$$
                We then need to prove by induction that $fracsqrt pi cdot n!(n+frac12)! > sqrt4$ for positive $n$.



                Yes. This is a lot of hard work. You could also just take the approach that was posted by Bernard and observe that $4x-x^2$ has a global maximum on the interval $[0,4]$ (the begin and endpoint are zeros of this polynomial), this maximum by symmetry occurs at $x=2$. The function has maximum value $8-4=4$. Observe then by the estimation lemma that $M =max_x in [0,4]((4x-x^2)^n)=4^n$, and the length of our path is $4$. We get:
                $$ int _x=0^4 f_n dx leq M L = 4^n cdot 4 = 4^n+1 $$



                Also see: ML-inequality for real integrals






                share|cite|improve this answer









                $endgroup$



                Below is a suggestion for a very tedious route, but I wanted to post it anyway. This is probably not how the question is meant to be done.



                Observe how by the binomial theorem we have that:
                $$I=int _x=0^4:f_n(x),dx =int _x=0^4: (4x-x^2 )^n,dx =int _x=0^4: sum_k=0^n binomnk (4x)^k cdot (-x^2)^n-k,dx$$
                We make this look a bit prettier:
                $$I=int _x=0^4: sum_k=0^n binomnk 4^k cdot (-1)^n-kx^2n-k,dx$$



                We may switch the order of integration and summation since this concerns polynomials (what I mean is that they are of finite order), we compute:
                $$I=sum_k=0^n binomnk frac12n-k+1 4^k cdot (-1)^n-k4^2n-k+1-0$$
                We collect some terms:
                $$I=sum_k=0^n binomnk frac12n-k+1 4^2n+1 cdot (-1)^n-k$$
                Using combinatorial formulae and some help from Wolfram Alpha, this can be exactly computed to be:
                $$ I =fracsqrt pi cdot n! cdot 2^2n+1(n+frac12)!=fracsqrt pi cdot n!(n+frac12)! cdot 4^n + frac12$$
                We then need to prove by induction that $fracsqrt pi cdot n!(n+frac12)! > sqrt4$ for positive $n$.



                Yes. This is a lot of hard work. You could also just take the approach that was posted by Bernard and observe that $4x-x^2$ has a global maximum on the interval $[0,4]$ (the begin and endpoint are zeros of this polynomial), this maximum by symmetry occurs at $x=2$. The function has maximum value $8-4=4$. Observe then by the estimation lemma that $M =max_x in [0,4]((4x-x^2)^n)=4^n$, and the length of our path is $4$. We get:
                $$ int _x=0^4 f_n dx leq M L = 4^n cdot 4 = 4^n+1 $$



                Also see: ML-inequality for real integrals







                share|cite|improve this answer












                share|cite|improve this answer



                share|cite|improve this answer










                answered 8 hours ago









                Wesley StrikWesley Strik

                2,310424




                2,310424



























                    draft saved

                    draft discarded
















































                    Thanks for contributing an answer to Mathematics Stack Exchange!


                    • Please be sure to answer the question. Provide details and share your research!

                    But avoid


                    • Asking for help, clarification, or responding to other answers.

                    • Making statements based on opinion; back them up with references or personal experience.

                    Use MathJax to format equations. MathJax reference.


                    To learn more, see our tips on writing great answers.




                    draft saved


                    draft discarded














                    StackExchange.ready(
                    function ()
                    StackExchange.openid.initPostLogin('.new-post-login', 'https%3a%2f%2fmath.stackexchange.com%2fquestions%2f3247910%2ffinding-the-constrain-of-integral%23new-answer', 'question_page');

                    );

                    Post as a guest















                    Required, but never shown





















































                    Required, but never shown














                    Required, but never shown












                    Required, but never shown







                    Required, but never shown

































                    Required, but never shown














                    Required, but never shown












                    Required, but never shown







                    Required, but never shown







                    Popular posts from this blog

                    ParseJSON using SSJSUsing AMPscript with SSJS ActivitiesHow to resubscribe a user in Marketing cloud using SSJS?Pulling Subscriber Status from Lists using SSJSRetrieving Emails using SSJSProblem in updating DE using SSJSUsing SSJS to send single email in Marketing CloudError adding EmailSendDefinition using SSJS

                    Кампала Садржај Географија Географија Историја Становништво Привреда Партнерски градови Референце Спољашње везе Мени за навигацију0°11′ СГШ; 32°20′ ИГД / 0.18° СГШ; 32.34° ИГД / 0.18; 32.340°11′ СГШ; 32°20′ ИГД / 0.18° СГШ; 32.34° ИГД / 0.18; 32.34МедијиПодациЗванични веб-сајту

                    19. јануар Садржај Догађаји Рођења Смрти Празници и дани сећања Види још Референце Мени за навигацијуу